The Student Room Group

The hard integral thread.

Scroll to see replies

Given In=0axn+0.5axdx\displaystyle I_n = \int_{0}^{a}x^{n+0.5}\sqrt{a-x} \mathrm dx, nZn \in \mathbb{Z}, n0n \geqslant 0

Show that In=(a4)n(2n+2n)I0n+1\displaystyle I_n = \Big(\frac{a}{4}\Big)^n \begin{pmatrix}2n+2 \\ n \end{pmatrix}\frac{I_0}{n+1}, n1n \geqslant 1

Determine the value of 02x104x2dx\displaystyle \int_{0}^{2}x^{10}\sqrt{4-x^2} \mathrm dx
(edited 6 years ago)
Original post by I hate maths
Given In=0axn+0.5axdx\displaystyle I_n = \int_{0}^{a}x^{n+0.5}\sqrt{a-x} \mathrm dx, nZn \in \mathbb{Z}, n0n \geqslant 0

Show that In=(a4)n(2n+2n)I0n+1\displaystyle I_n = \Big(\frac{a}{4}\Big)^n \begin{pmatrix}2n+2 \\ n \end{pmatrix}\frac{I_0}{n+1}, n1n \geqslant 1

Determine the value of 02x104x2dx\displaystyle \int_{0}^{2}x^{10}\sqrt{4-x^2} \mathrm dx


This reads too much like an FP2 question for my liking, nevertheless... :tongue:

Unparseable latex formula:

\displaystyle[br]\begin{align*}3I_n &= 2\left(n+\frac{1}{2}\right)\int_0^a x^{n-0.5}(a-x)\sqrt{a-x} \, \mathrm{d}x \\ &= (2n+1)\left[aI_{n-1} - I_n\right]\end{align*}



via IBP, then we have

Unparseable latex formula:

\displaystyle[br]\begin{align*}I_n &= \frac{2n+1}{2n+4}aI_{n-1} \\ &= \frac{8a^n(2n+1)!!}{(2n+4)!!}I_0 \\ & = \frac{8a^n(2n+2)!}{2^{n+1}(n+1)! \cdot 2^{n+2}(n+2)!} I_0 \\ & = \left(\frac{a}{4}\right)^n {{2n+2} \choose {n}} \frac{I_0}{n+1} \end{align*}



For the last integral, just use xx2x \mapsto x^2 to get

Unparseable latex formula:

\displaystyle [br]\begin{align*}\int_0^2 x^{10}\sqrt{4-x^2} \, \mathrm{d}x & = \frac{1}{2}\int_0^4 x^{4.5} \sqrt{4-x} \, \mathrm{d}x = \frac{1}{2} {{10} \choose 4} \frac{2\pi}{5} = 42\pi\end{align*}

Original post by Zacken
x


Work of a master craftsman... I've learned a lot from reading it like how to align stuff in the middle of the page, here let me try.

Unparseable latex formula:

\displaystyle[br][br]\begin{align*} \int_{1}^2 1 \mathrm{d}x &= 2-1\\ &=1\end{align*}



Omg it worked!!
This is very aesthetically pleasing.

Prove that

01xex(121x+1ex1)dx=112log2π\displaystyle \int_0^\infty \frac 1 {xe^x} \left(\frac 1 2 - \frac 1 x + \frac 1 {e^x - 1}\right) \, \mathrm dx = 1 - \frac 1 2 \log 2\pi

(consider expressions of the digamma function)
Original post by Indeterminate
This is a nice one :biggrin:

Evaluate

0 dxx(x2+kx+1)(1x+x2x3+(x)n)\displaystyle \int_{0}^{\infty} \dfrac{\ dx}{\sqrt{x}(x^2 + kx + 1)(1-x+x^2-x^3 + \cdots (-x)^n)}


Assuming that nn is even, this is I=0(1+x)dxx(x2+kx+1)(1+xn+1)I=\displaystyle \int_{0}^{\infty} \dfrac{(1+x)\mathrm dx}{\sqrt{x}(x^2 + kx + 1)(1+x^{n+1})} (if nn is odd, the integral does not converge.)

The substitution xx1x\mapsto x^{-1} gives I=0(1+x)xn+1dxx(x2+kx+1)(1+xn+1)I=\displaystyle \int_{0}^{\infty} \dfrac{(1+x)x^{n+1}\mathrm dx}{\sqrt{x}(x^2 + kx + 1)(1+x^{n+1})} .

Adding, 2I=0(1+x)dxx(x2+kx+1)2I=\displaystyle \int_{0}^{\infty} \dfrac{(1+x)\mathrm dx}{\sqrt{x}(x^2 + kx + 1)} .

Consider the substitution y=x1xy=\sqrt x-\dfrac1{\sqrt x}. This gives dy=1+x2xxdx\mathrm dy=\dfrac{1+x}{2x\sqrt x}\,\mathrm dx, so that yy increases monotonically from  to -\infty\textrm{ to }\infty as xx goes from 0 to 0\textrm{ to }\infty.

Also y2=x+1x2y^2=x+\dfrac1x-2.

Thus I=0(1+x)dx2xx(x+k+1x)=dyy2+k+2=πk+2I=\displaystyle \int_{0}^{\infty} \dfrac{(1+x)\mathrm dx}{2x\sqrt{x}\left(x + k + \dfrac1x\right)}=\int_{-\infty}^\infty \dfrac{\mathrm dy}{y^2+k+2}=\dfrac\pi{\sqrt{k+2}} .
Reply 1685
Original post by _gcx
This is very aesthetically pleasing.

Prove that

01xex(121x+1ex1)dx=112log2π\displaystyle \int_0^\infty \frac 1 {xe^x} \left(\frac 1 2 - \frac 1 x + \frac 1 {e^x - 1}\right) \, \mathrm dx = 1 - \frac 1 2 \log 2\pi

(consider expressions of the digamma function)
LHS Looks ugly but the RHS is pretty.
(edited 5 years ago)
Original post by _gcx
This is very aesthetically pleasing.

Prove that

01xex(121x+1ex1)dx=112log2π\displaystyle \int_0^\infty \frac 1 {xe^x} \left(\frac 1 2 - \frac 1 x + \frac 1 {e^x - 1}\right) \, \mathrm dx = 1 - \frac 1 2 \log 2\pi

(consider expressions of the digamma function)


In what follows assume that t>0t>0, that all integrals converge, and that all limits taken are valid.

Start with the Abramowitz & Stegun formula for the digamma function ψ(t)=Γ(t)Γ(t)=0[exxetx1ex]dx\psi(t)=\dfrac{\Gamma' (t)}{\Gamma (t)}=\displaystyle \int_0^\infty\left[\dfrac{\mathrm e^{-x}}x-\dfrac{\mathrm e^{-tx}}{1-\mathrm e^{-x}}\right]\mathrm dx.

We also need the well-known identities 1t=0etxdx\dfrac1t=\displaystyle \int_0^\infty \mathrm e^{-tx}\mathrm dx and lnt=0exetxxdx\ln t=\displaystyle \int_0^\infty \dfrac{\mathrm e^{-x}-\mathrm e^{-tx}}x\mathrm dx.

Thus ψ(t+1)lnt12t=0[etxxe(t+1)x1exetx2]dx\psi(t+1)-\ln t-\dfrac1{2t}=\displaystyle \int_0^\infty\left[\dfrac{\mathrm e^{-tx}}x-\dfrac{\mathrm e^{-(t+1)x}}{1-\mathrm e^{-x}}-\dfrac{\mathrm e^{-tx}}2\right]\mathrm dx

=0etx[1x1ex112]dx{}=\displaystyle \int_0^\infty\mathrm e^{-tx}\left[\dfrac1x-\dfrac1{\mathrm e^x-1}-\dfrac12\right]\mathrm dx. Now, integrating both sides with respect to tt leads to

lnΓ(t+1)tlnt+t12lnt+c=0etxx[121x+1ex1]dx\ln\Gamma(t+1)-t\,\ln t+t-\frac12\ln t+c=\displaystyle \int_0^\infty\dfrac{\mathrm e^{-tx}}x\left[\dfrac12-\dfrac1x+\dfrac 1{\mathrm{e}^x-1}\right]\mathrm dx, where cc is a constant. (*)

According to Stirling's theorem,

limtΓ(t+1)tt+1/2et=2π\lim\limits_{t\to\infty}\dfrac{ \Gamma(t+1)}{t^{t+1/2}\mathrm e^{-t}}=\sqrt{2\pi}, so that limt[lnΓ(t+1)(t+12)lnt+t]=12ln2π\lim\limits_{t\to\infty}[\,\ln \Gamma(t+1)-(t+\frac12)\ln t+t\,]=\frac12\ln 2\pi.

Letting tt tend to infinity in (*) thus gives 12ln2π+c=0\frac12\ln 2\pi+c=0.

Finally, putting t=1t=1 in (*) gives 01xex(121x+1ex1)dx=1+c=112ln2π\displaystyle \int_0^\infty \frac 1{x\mathrm e^x} \left(\frac 12 - \frac 1x + \frac 1{\mathrm e^x - 1}\right) \, \mathrm dx = 1 +c=1-\textstyle\frac 12 \ln 2\pi.
Original post by ciberyad
In what follows assume that t>0t>0, that all integrals converge, and that all limits taken are valid.

Start with the Abramowitz & Stegun formula for the digamma function ψ(t)=Γ(t)Γ(t)=0[exxetx1ex]dx\psi(t)=\dfrac{\Gamma' (t)}{\Gamma (t)}=\displaystyle \int_0^\infty\left[\dfrac{\mathrm e^{-x}}x-\dfrac{\mathrm e^{-tx}}{1-\mathrm e^{-x}}\right]\mathrm dx.

We also need the well-known identities 1t=0etxdx\dfrac1t=\displaystyle \int_0^\infty \mathrm e^{-tx}\mathrm dx and lnt=0exetxxdx\ln t=\displaystyle \int_0^\infty \dfrac{\mathrm e^{-x}-\mathrm e^{-tx}}x\mathrm dx.

Thus ψ(t+1)lnt12t=0[etxxe(t+1)x1exetx2]dx\psi(t+1)-\ln t-\dfrac1{2t}=\displaystyle \int_0^\infty\left[\dfrac{\mathrm e^{-tx}}x-\dfrac{\mathrm e^{-(t+1)x}}{1-\mathrm e^{-x}}-\dfrac{\mathrm e^{-tx}}2\right]\mathrm dx

=0etx[1x1ex112]dx{}=\displaystyle \int_0^\infty\mathrm e^{-tx}\left[\dfrac1x-\dfrac1{\mathrm e^x-1}-\dfrac12\right]\mathrm dx. Now, integrating both sides with respect to tt leads to

lnΓ(t+1)tlnt+t12lnt+c=0etxx[121x+1ex1]dx\ln\Gamma(t+1)-t\,\ln t+t-\frac12\ln t+c=\displaystyle \int_0^\infty\dfrac{\mathrm e^{-tx}}x\left[\dfrac12-\dfrac1x+\dfrac 1{\mathrm{e}^x-1}\right]\mathrm dx, where cc is a constant. (*)

According to Stirling's theorem,

limtΓ(t+1)tt+1/2et=2π\lim\limits_{t\to\infty}\dfrac{ \Gamma(t+1)}{t^{t+1/2}\mathrm e^{-t}}=\sqrt{2\pi}, so that limt[lnΓ(t+1)(t+12)lnt+t]=12ln2π\lim\limits_{t\to\infty}[\,\ln \Gamma(t+1)-(t+\frac12)\ln t+t\,]=\frac12\ln 2\pi.

Letting tt tend to infinity in (*) thus gives 12ln2π+c=0\frac12\ln 2\pi+c=0.

Finally, putting t=1t=1 in (*) gives 01xex(121x+1ex1)dx=1+c=112ln2π\displaystyle \int_0^\infty \frac 1{x\mathrm e^x} \left(\frac 12 - \frac 1x + \frac 1{\mathrm e^x - 1}\right) \, \mathrm dx = 1 +c=1-\textstyle\frac 12 \ln 2\pi.


Brilliant!
Ah, this thread was so much fun!
Original post by Kummer
Ah, this thread was so much fun!


The legend himself :redface:
Original post by Kummer
Ah, this thread was so much fun!


Good days! You just finished second year at Uni?
Original post by _gcx
The legend himself :redface:
I agree, Ernst Kummer is such a legend! :wink: I mean what's not to like: he wedded algebra and number theory to give us a beautiful baby known as algebraic number theory!
Need to keep this thread alive.

A sum that took me a bit of time to get:

n=1arctan(18n2)\displaystyle \sum_{n = 1}^\infty \arctan \left({\frac 1 {8n^2}}\right)

(think of expressions for sine)

And of course some fun with Feynman's trick:

01arctanxx1x2dx\displaystyle \int_0^1 \frac{\arctan x} {x \sqrt{1 - x^2}} \mathrm dx
arctg(x*x)
Original post by _gcx
A sum that took me a bit of time to get:

n=1arctan(18n2)\displaystyle \sum_{n = 1}^\infty \arctan \left({\frac 1 {8n^2}}\right)

(think of expressions for sine)

Liked this one - when you say sine expression, guess you mean Euler's infinite product if we did it the same way.
Original post by RichE
Liked this one - when you say sine expression, guess you mean Euler's infinite product if we did it the same way.


Yeah same method :smile: Would be interested if there were an elementary way to evaluate it.
Original post by _gcx
Yeah same method :smile: Would be interested if there were an elementary way to evaluate it.


You could probably work out the sum of 8n^2/(x^2+64n^4) using complex analysis, and then integrate your answer.
Original post by _gcx
Need to keep this thread alive.

A sum that took me a bit of time to get:

n=1arctan(18n2)\displaystyle \sum_{n = 1}^\infty \arctan \left({\frac 1 {8n^2}}\right)

(think of expressions for sine)

Start with the identities

cotx=1x+n=12xx2n2π2\cot x=\dfrac1x+\displaystyle\sum_{n=1}^\infty \dfrac{2x}{x^2-n^2\pi^2}, and cothx=1x+n=12xx2+n2π2\coth x=\dfrac1x+\displaystyle\sum_{n=1}^\infty \dfrac{2x}{x^2+n^2\pi^2},

so that

πcotπxπcothπx=n=1(2xx2n22xx2+n2)=n=14n2xx4n4\pi\cot \pi x-\pi\coth \pi x=\displaystyle\sum_{n=1}^\infty \left(\dfrac{2x}{x^2-n^2}-\dfrac{2x}{x^2+n^2} \right)=\sum_{n=1}^\infty \dfrac{4n^2x}{x^4-n^4}.

Then

πcotπ(1+i)xπcothπ(1+i)x=(1+i)n=14n2x4x4+n4\pi\cot \pi(1+\mathrm i)x-\pi\coth \pi(1+\mathrm i)x=-(1+\mathrm i) \displaystyle\sum_{n=1}^\infty \dfrac{4n^2x}{4x^4+n^4}. (*)

For simplicity, let u=tanπxu=\tan \pi x and v=tanhπxv=\tanh \pi x.

Then

cotπ(1+i)xcothπ(1+i)x=1iuvu+iv1+iuvv+iu\cot \pi(1+\mathrm i)x-\coth \pi(1+\mathrm i)x=\dfrac{1-\mathrm iuv}{u+\mathrm iv}-\dfrac{1+\mathrm iuv}{v+\mathrm iu}

=(1+i)uvuv(u+v)u2+v2=(1+i)u(1v2)v(1+u2)u2+v2=(1+\mathrm i)\dfrac{u-v-uv(u+v)}{u^2+v^2}=(1+\mathrm i)\dfrac{u(1-v^2)-v(1+u^2)}{u^2+v^2}

=(1+iπ)udvdxvdudxu2+v2=(1+iπ)ddxarctanvu=\left(\dfrac{1+\mathrm i}\pi\right) \dfrac{u\dfrac{\mathrm dv}{\mathrm dx}-v\dfrac{\mathrm du}{\mathrm dx}}{u^2+v^2}=\left( \dfrac{1+\mathrm i}\pi \right) \dfrac{\mathrm d}{\mathrm dx}\arctan \dfrac vu. (**)

Therefore, from (*) and (**),

n=14n2x4x4+n4=π1+i(cotπ(1+i)xπcothπ(1+i)x)\displaystyle\sum_{n=1}^\infty \dfrac{4n^2x}{4x^4+n^4}=-\dfrac{\pi}{1+\mathrm i}(\cot \pi(1+\mathrm i)x-\pi\coth \pi(1+\mathrm i)x)

=ddxarctan(tanhπxtanπx)=-\dfrac{\mathrm d}{\mathrm dx}\arctan \left(\dfrac{\tanh \pi x}{\tan \pi x} \right).

Note also that 4n2xn4+4x4=ddxarctan(2x2n2)\dfrac{4n^2x}{n^4+4x^4}= \dfrac{\mathrm d{}}{\mathrm dx} \arctan \left( \dfrac{2x^2}{n^2} \right).

Therefore ddxn=1arctan(2x2n2)=ddxarctan(tanhπxtanπx)\dfrac{\mathrm d}{\mathrm dx} \displaystyle \sum_{n=1}^\infty \arctan \left( \dfrac{2x^2}{n^2} \right)=-\dfrac{\mathrm d}{\mathrm dx} \arctan \left( \dfrac{\tanh \pi x}{\tan \pi x} \right).

Thus n=1arctan(2x2n2)=arctan(tanhπxtanπx)+C\displaystyle\sum_{n=1}^\infty \arctan \left( \dfrac{2x^2}{n^2} \right)=-\arctan \left( \dfrac{\tanh \pi x}{\tan \pi x} \right)+C.

Letting x0x\to 0, we have 0=arctan1+C0=-\arctan 1+C, so C=π4C=\dfrac\pi4.

Thus n=1arctan(2x2n2)=π4arctan(tanhπxtanπx)\displaystyle\sum_{n=1}^\infty \arctan \left( \dfrac{2x^2}{n^2} \right)=\dfrac\pi4-\arctan \left( \dfrac{\tanh \pi x}{\tan \pi x} \right),

which is valid for 0x10 \leq x \leq 1 (where RHS is continuous.)

Putting x=14x=\dfrac14, we get n=1arctan(18n2)=π4arctan(tanhπ4)\displaystyle\sum_{n=1}^\infty \arctan \left(\dfrac1{8n^2}\right)={} \dfrac{\pi}{4}-\arctan \left(\tanh \dfrac\pi4 \right).
(edited 5 years ago)
Original post by Dabo_26
integral easy lol.png
that was my FP2 prep a few days ago lol


Oof lol
Can't wait for Further Maths AS A-Level next year :biggrin:

Quick Reply

Latest